Two rods, Two hinges.

The setup shown consists of two identical rods (mass m) hinged together at the ends.
This setup is held in the position shown.
Now we start applying a constant force F to the free end as shown
Find the initial angular acceleration of the rods.

39 Answers

24
eureka123 ·

I didnt get the ques...

even if right side rod moves..what effect will it have on left side rod ??

49
Subhomoy Bakshi ·

yeah α will hav equal magnitude since the common hinge 4 both comes down with fixed acceleration........trying to figure out the soln nw..........

49
Subhomoy Bakshi ·

@Philip..wats the answer...i'll post soln only if i get correct answer..

49
Subhomoy Bakshi ·

nishant bhaiya plz dont post answer..i'll try out and do it...

1
Philip Calvert ·

no. i will not post the answer rite now.
i posted it here

http://targetiit.com/iit-jee-forum/posts/round-and-round-and-snap-12357.html

and see the result. Its gone dead...

1
injun joe ·

Is no one paying attention to this??

24
eureka123 ·

i tried it in varoius ways..but found that soln is this much only

taking torque abt COM of right rod,
τ=ICentroidal.αright
F(Lsinθ2)=mL212.αright
=> αright=6FsinθmL

plz tell me if it is rite or wrong....

1
Philip Calvert ·

why are you all neglecting the forces at the hinge....
and the force of gravity

24
eureka123 ·

ya...i missed forces of hinge....but why forces of gravity ??they wont produce torque abt CM

Your Answer

Close [X]